Contracts & Sales Missed MBE Questions

Ace your homework & exams now with Quizwiz!

A groom left his bride at the alter on the day of her wedding. The bride could not bear to keep any painful reminders of the occasion, so she offered to sell her wedding dress to one of her bridesmaids for $5,000. The bride stated that the offer would remain open for 30 days. The bridesmaid said that she was interested but would have to think about it. A week later, the bridesmaid emailed the bride to ask if the price included a custom-made veil that the bride had worn. The bride did not respond to the bridesmaid's question. Within the 30-day period, the bridesmaid accepted the bride's initial offer of $5,000 for the wedding dress. In response, the bride stated that the bridesmaid could only buy the wedding dress fo $6,000. Was a contract formed when the bridesmaid accepted the initial offer of $5,000? Answers: A. Yes, because the bridesmaid's question did not constitute a counteroffer. B. Yes, because the bride was required to keep the initial offer open for the 30-day period. C. No, because the bridesmaid's question acted as a counteroffer and rejection of the $5,000 offering price. D. No, because the bride raised the price of the dress to $6,000.

Answer choice A is correct. A counteroffer is an offer made by an offeree to the offeror related to the same matter as the original offer and proposing a substituted bargain differing from that proposed by the original offer. An offeree's power of acceptance is terminated by the making of a counteroffer. A counteroffer acts as a rejection of the original offer and creates a new offer. However, mere suggestions or inquiries, including requests for clarification or statements of intent made in a response by the offeree, do not constitute a counteroffer. Here, the bridesmaid asked if the $5,000 price included the custom-made veil. This question was merely a request for clarification as to what was included in the offer; it was not a counteroffer. Therefore, the bridesmaid could accept the original offer of $5,000 for the wedding dress within the 30-day period. Answer choice B is incorrect because the bride was not required to keep the offer open for 30 days. In general, an offer can be revoked by the offeror at any time prior to acceptance. An offer is revoked when the offeror makes a manifestation of an intention not to enter into the proposed contract. In this case, the bride could have revoked the offer at any time prior to the expiration of 30 days, provided that she communicated the revocation to the bridesmaid. Importantly, the bridesmaid did not provide consideration to keep the offer open and the offer was not otherwise irrevocable. Answer choice C is incorrect because, as stated above, the bridesmaid's question about the custom-maid veil was merely a request for clarification, not a counteroffer. Thus, it did not constitute a rejection. Answer choice D is incorrect because although the bride could have raised the price of the wedding dress from $5,000 to $6,000, she did not attempt to do so until after the bridesmaid accepted the original offer and created an enforceable contract.

On May 10, the coach of a youth league baseball team sent a letter to a supplier asking the supplier to promptly ship 20 red jerseys to him. On May 15, the supplier received this letter and sent the coach a reply letter accepting the offer. On May 16, the supplier realized that he had no red jerseys with which to fill the order, and sends the coach 20 blue jerseys with a note that the blue jerseys were tendered as an accommodation. The coach received the jerseys and accommodation note on May 18, and received the supplier's acceptance letter on May 19. On May 20, which of the following is a correct statement of the parties' legal rights and duties? Answers: A. The coach can either accept or reject the blue jerseys and, in either event, recover damages, if any, for breach of contract. B. The coach can either accept or reject the blue jerseys, but if he rejects them, he will thereby waive any remedy for breach of contract. C. The supplier's shipment of nonconforming goods constituted an acceptance of the coach's offer, thereby creating a contract for the sale of the blue jerseys. D. The supplier's shipment of the blue jerseys constituted a counteroffer.

Answer choice A is correct. A seller's shipment of nonconforming goods with a notice of accommodation does not constitute an acceptance and breach, but rather a counteroffer, which the buyer is free to either accept or reject. However, an offer calling for prompt shipment can be accepted either by a prompt promise to ship or by the prompt shipment of goods. Under the mailbox rule, an acceptance is effective when mailed. Consequently, on May 15 when the supplier mailed his acceptance to the coach, a contract for 20 red jerseys was formed, even though the coach did not receive this acceptance until May 19. With regard to the May 16 shipment of blue jerseys by the supplier that the coach received on May 18, the coach may accept or reject these jerseys as nonconforming goods and, in either event, recover damages, if any. Answer choice B is incorrect. Although a seller's shipment of nonconforming goods with a notice of accommodation does not constitute an acceptance and breach, but rather a counteroffer, which the buyer is free to either accept or reject, this is only true if the seller has not already accepted the buyer's offer. Here, the supplier had earlier mailed its acceptance of the coach's offer to supply red jerseys, so the supplier's shipment of the blue jersey constitutes a breach of contract. Consequently, the coach may reject the blue jerseys without waiving any remedy for breach of contract. Answer choice C is incorrect because the supplier's shipment of the blue jerseys did not constitute an acceptance of the coach's offer, but a breach of the contract that had already been formed for the shipment of red jerseys. Answer choice D is incorrect because, although typically an accommodation shipment constitutes a counteroffer rather than an acceptance and breach of the contract, here the shipment, despite being designated as an accommodation, cannot be a counteroffer because the supplier had already accepted the coach's offer.

A collector of rare cars sent a letter to a well-known car enthusiast on January 2 regarding the possible sale by the collector of a limited-edition car. on January 3, the car enthusiast sent a letter to the collector, offering to purchase the limited-edition car for a price of $35,000. The letter also stated that the car enthusiast reserved the right to cancel the contract before January 10. Upon receiving the car enthusiast's offer on January 5, the collector immediately mailed an acceptance of the offer to the car enthusiast. on January 6, the car enthusiast sent a fax to the collector cancelling the contract. The car enthusiast received the collector's acceptance on January 7. The collector sued the car enthusiast for breach of contract. Which party is likely to prevail? Answers: A. The car enthusiast, because his letter on January 3 was not a valid offer. B. The car enthusiast, because the collector's acceptance arrived after he cancelled the contract. C. The collector, because the car enthusiast's offer was irrevocable. D. The collector, because his acceptance was effective on January 5.

Answer choice A is correct. A statement is an offer only if the person to whom it is communicated could reasonably interpret it as an offer. It must express the present intent of a person to be legally bound to a contract. The primary test of whether a communication is an offer is whether an individual receiving the communication would believe that he could enter into an enforceable deal by satisfying the condition. In his letter to the collector offering to purchase the car, the car enthusiast reserved the right to cancel the contract before January 10. Accordingly, it would not be reasonable for the collector to believe that he could form an enforceable deal with the car enthusiast by accepting the offer prior to January 10. Therefore, the collector's attempted acceptance was not effective because there was not a valid offer. Answer choice B is incorrect. Under the mailbox rule, even if the offer had been valid, the collector's acceptance would have been effective upon mailing, not receipt. Here, the fact that the car enthusiast cancelled the contract prior to receiving the collector's acceptance has no effect on the outcome because there was no valid offer to accept. Answer choice C is incorrect because there was no consideration to make the offer irrevocable. Moreover, the car enthusiast specifically reserved the right to revoke the offer (cancel the contract). Answer choice D is incorrect. Although the collector's acceptance would have been effective when it was mailed on January 5 under the mailbox rule, there was not a valid offer for the collector to accept at that time.

A builder ordered 100 squares of shingles from a home-supply store for installation on the roofs of homes that he was building. The builder agreed to a price of $120 per square. Delivery was set for no later than noon on the following Monday to the construction site. The truck from the store with the ordered shingles arrived at 1:00 pm the following Monday. The builder rejected the shipment due to its failures to arrive on time. The store, which regularly sold 600 squares of shingles per week, resold the squares that had been rejected by the builder, at a price of $110 per square. the store would have made a profit of $3,000 has the builder accepted the shingles. If the store sues the builder for breach of contract, how much can the store recover from the builder? Answers: A. Nothing B. $1,000, the contract price minus the resale price C. $3,000, the store's lost profit on the initial sale. D. $4,000, to recover the store's total expectation damages.

Answer choice A is correct. Because the builder did not breach the contract, the builder is not liable to the store for damages. The sale of the shingles is a sale of goods and therefore is governed by the Uniform Commercial Code (UCC), even though the builder planned to incorporate the shingles into homes being constructed. Under the UCC, a buyer may reject the goods if the seller fails to make a perfect tender, which includes a timely delivery of the goods. Unlike under the common law, substantial performance of a contractual obligation is not sufficient. Answer choice B is incorrect. Although this is one measure of expectation damages of a seller of goods, the store is not entitled to damages because the builder as buyer was entitled to a perfect tender of goods, which the store failed to make by its late delivery of the shingles. Answer choice C is incorrect. Lost profits are a measure of a seller's expectation damages if the seller is a lost-volume seller, as the store in this case is with regard to the shingles. This remedy would only be available to the store had the builder breached the contract, but the builder did not. Answer choice D is incorrect because, even if the store had been entitled to expectation damages, it could not recoup both measures of its expectation damages.

A manufacturer of t-shirts contracted with a new clothing store to sell the store 1,000 t-shirts per month for a period of two years. The clothing store's signature color for their clothing was an orange-tinted red color, called coquelicot, which is very difficult to replicate on a consistent basis. The contract specified that any t-shirts that were not coquelicot could be returned, but it was silent with regard to the return of any t-shirts for other reasons. One year into the contract, the store decided to switch to coquelicot-colored baseball hats instead of t-shirts. The store returned the most recent shipment of t-shirts to the manufacturer and demanded a refund. The manufacturer refused to grant the refund, and the store sued the manufacturer for damages. At trial, the manufacturer introduced the the contract, which clearly stated that t-shirts that were not coquelicot could be returned. The store then attempted to introduce evidence that it had returned t-shirts for other reasons to the manufacturer in the past and received a refund. Is this evidence admissible? Answers A. Yes, because the evidence is relevant to show that the manufacturer had accepted the return of coquelicot-colored t-shirts in the past. B. Yes, because the evidence can reasonably establish the parties' course of dealing on this issue. C. No, because the evidence regarding the return of the t-shirts violates the parol evidence rule. D. No, because the express term in the contract regarding the return of t-shirts takes precedence over the course-of-performance.

Answer choice A is correct. Course-of-performance evidence is admissible under the UCC to explain or supplement a contract. A course of performance is a sequence of conduct that is relevant to understanding an agreement between the parties if: (i) the agreement involves repeated occasions for performance by a party, and (ii) the other party accepts performance without objection and with knowledge of the course of performance. Here, there were monthly purchases of t-shirts and evidence that the manufacturer had accepted returns of coquelicot-colored t-shirts in the past. Therefore, this course-of-performance evidence is admissible. Answer choice B is incorrect because the course of dealing refers to conduct between the parties based on another contractual relationship. Here, because there is only the one contract between the two parties and the evidence relates to the performance of that contract, rather than another contract, the evidence does not relate to their course of dealing, but rather their course of performance. Answer choice C is incorrect because the parol evidence rule does not prevent the introduction of evidence regarding conduct by the parties after the written contract was executed. Answer choice D is incorrect because the agreement was silent as to the return of the colquelicot-colored t-shirts. Thus, course-of-performance evidence is admissible to explain how the parties viewed the right of the store to return such t-shirts.

A tire dealer phoned in an order to a wholesaler for various tires of specific sizes and types at a total cost of $25,000. The wholesaler sent the dealer an acceptance of the order that listed the quantities of the various sizes and types of tires that the dealer had ordered. The acceptance identified the wholesaler in its heading, and in its body it identified the dealer as the person who had ordered the tires. Twelve days after the dealer received the acceptance but prior to the wholesaler's shipment of the tires, the dealer objected to the order. Unable to resell the tires for $25,000, the wholesaler sued the dealer for damages. The dealer asserted the Statute of Frauds as a defense. Will the dealer be successful in its defense against the wholesaler's action? Answers: A. No, because the dealer failed to timely object to the written acceptance. B. No, because there is no need for a writing when both parties to a contract for the sale of goods are merchants. C. Yes, because the dealer never signed a writing that identified it as the buyer of the tires. D. Yes, because the dealer objected to the written acceptance before the wholesaler had shipped the tires.

Answer choice A is correct. If both parties are merchants and a memorandum sufficient against one party is sent to the other party, who has reason to know its contents, and the receiving party does not object in writing within 10 days, then the contract is enforceable against the receiving party even though he has not signed it. Here, the dealer did not object to the written acceptance sent by the wholesaler within 10 days. Therefore, the contract as expressed in the written acceptance is enforceable against the dealer despite the absence of the dealer's signature. Answer choice B is incorrect because the UCC does not completely eliminate the need for a writing when both parties are merchants. Answer choice C is incorrect. Generally, the Statute of Frauds requires that there be a writing signed by the party against whom the contract is to be enforced. However, there is an exception when both parties are merchants if a memorandum sufficient against one party is sent to the other party, who has reason to know of its contents, and the receiving party does not object in writing within 10 days. Answer choice D is incorrect. Under the special memorandum provision of the UCC Statute of Frauds that is applicable when both parties are merchants, a party who receives a memorandum of the contract from the other party has 10 days to object in writing to the memorandum, or the party receiving the memorandum waives the Statute of Frauds defense to the contract.

While attending a rodeo on August 20, a hat maker entered into a valid, written agreement with the rodeo manager to make 500 leather cowboy hats for an upcoming rodeo event at a price of $75 per hat. Per the agreement, the rodeo manager agreed to pay one-fourth of the total purchase price to a tannery owner to whom the hat maker owed a debt for a previous leather order. On August 25, the hat maker changed his mind about paying one-fourth of the purchase price to the tannery owner. The hat maker and rodeo manager subsequently executed a valid modification of the original agreement. The rodeo manager's brother was also present on August 20 when the original agreement was executed, but he did not know about the August 25 modification of the agreement to no longer pay the tannery owner. On August 30, the brother, who was friends with the tannery owner, called and told him that his debt from the hat maker would finally be paid off. However, the rodeo manager refused to pay one-fourth of the purchase price to the tannery owner. If the tannery owner sues the rodeo manager for one-fourth of the purchase price, will he recover? Answers: A. No, because the tannery owner did not rely on the August 20 agreement between the hat maker and the rodeo manager. B. No, because there was no consideration for the promise to pay the tannery owner by the hat maker and the rodeo manager. C. Yes, because the tannery owner had the right to sue the rodeo manager to enforce the contract between the rodeo manager and the hat maker. D. Yes, because the rodeo manager agreed to pay one-fourth of the purchase price to the tannery owner on August 20.

Answer choice A is correct. If performance of a promise would satisfy an actual, supposed, or asserted duty of the promisee to a third party, and the promisee did not intend to make a gift to the third party, then the third party is an intended beneficiary who is a creditor beneficiary. A creditor beneficiary has the right to sue either the promisor or the promisee to enforce the contract. Here, the tannery owner was a creditor beneficiary as of August 20. However, the hat maker and the rodeo manager agreed on August 25 to not pay the tannery owner. The tannery owner did not know about the August 20 agreement until August 30, after the two parties had agreed to not pay the tannery owner. Thus, he did not rely upon the original agreement that was later modified by both parties. For this reason, answer choices C and D are incorrect. Answer choice B is incorrect because an agreement to benefit a third party does not need to be supported by separate consideration. In addition, the tannery owner cannot recover because the two parties to the original agreement eliminated the third-party beneficiary agreement on August 25. Because the original agreement involved the sale of goods, no consideration is required to amend it, as long as the amendment is done in good faith.

A school's sports director entered into a written contract to purchase jerseys for the school's track team for a total cost of $9,000. The school mascot was to be imprinted on the front of each jersey. After the seller had acquired the jerseys but before they had been imprinted, the director emailed the seller and requested, in good faith, that the logo of the team's sponsor be imprinted on the back of the jerseys at no additional cost. In a reply email, the seller agreed to the director's request. When the jerseys arrived at the school, only the mascot, not the sponsor's logo, appeared on the jerseys. The sports director rejected the jerseys and refused to pay for them. The seller sued the school for breach of contract. Who will prevail? Answers: A. The director, because of the perfect tender rule. B. The director, because the director was not a merchant. C. The seller, because the jerseys were specially manufactured goods. D. The seller, because the seller did not receive consideration for the modification.

Answer choice A is correct. In a sale-of-goods transaction, the Uniform Commercial Code (UCC) requires that the goods tendered by the seller conform in all respects to the contract. Although the shipped jerseys conformed to the specifications of the original contract, this contract was modified by the subsequent emails between the director and the seller. Answer choice B is incorrect because the school's status as a non-merchant is irrelevant to determining the terms of this contract or its breach. Answer choice C is incorrect because the fact that the jerseys were specially manufactured goods is relevant only when the buyer is claiming the statute of frauds as a defense. The fact that the goods were specially manufactured is irrelevant to the validity of the emailed agreement to modify the original contract in this case. Answer choice D is incorrect because the UCC, rejecting the preexisting duty rule, does not require consideration in order for the good-faith modification of a contract to be valid.

A life coach entered into a contract with an introverted woman to provide a week-long course that served as an introduction to the life coach's year-long program. Under the terms of the contract, the life coach would work with the woman full-time for a week to start teaching her to interact with people with more confidence. The woman would then participate in a speed-dating event as her "graduation" from the course. The contract also provided that unless the woman received an offer for a second date at the speed-dating event, she had no obligation to pay for the introductory course. If she received an offer for a second date at the event, she would be required to pay the life coach $1,000 within three days of the speed-dating event. After working with the life coach pursuant to the terms of the contract, the woman attended the speed-dating event. Four days after the event, when the woman still had not paid the life coach, the life coach brought a breach of contract action against the woman to recover $1,000 under the contract. Which of the following best states the burden of proof that will apply in this case? Answers: A. Because the contract involved a condition precedent, the life coach must prove that the woman received an offer of a second date at the speed-dating event to recover. B. Because the contract involved a condition precedent, the woman must prove that she did not receive an offer of a second date at the speed-dating event to avoid liability. C. Because the contract involved a condition subsequent, the life coach must prove that the woman received an offer of a second date at the speed-dating event to recover. D. Because the contract involved a condition subsequent, the woman must prove that she did not receive an offer of a second date at the speed-dating event to avoid liability.

Answer choice A is correct. Performance by one or both of the parties may be made expressly conditional in the contract, and the conditions may precede the obligation to perform (condition precedent), or may discharge the duty to perform after a particular event occurs (condition subsequent). Here, the condition that the woman must receive an offer for a second date at the speed-dating event preceded her obligation to pay the life coach. If she received no offer, no duty to pay attaches. Therefore, it is a condition precedent. If a defendant's duty is subject to a condition precedent, then the plaintiff (here, the life coach) has the burden of proving that the condition occurred in order to recover. Therefore, answer choice A correctly states the burden of proof in this case. Answer choice B is incorrect because it states the burden of proof for cases when the defendant's duty is subject to a condition subsequent. Answer choices C and D are incorrect because they incorrectly state that the contract included a condition subsequent. Here, unless the woman received an offer for a second date at the speed-dating event, she had no obligation to pay for the introductory course. Therefore, because no obligation to perform attaches unless and until the woman received an offer for a second date at the speed-dating event, this was a condition precedent.

A salon owner contacted a manufacturer by email about purchasing shampoo sinks. The manufacturer sent the salon owner the following email: "I will sell you four shampoo sinks at a discounted price of $300 apiece." The salon owner responded immediately, rejecting the offer. However, due to a transmission problem in the internet routing system, the message was not delivered to the manufacturer until the following day. In the meantime, the salon owner contacted several other sellers, all of whom made significantly higher offers. The salon owner then sent another email to the manufacturer, stating, "I accept your offer." This email was delivered immediately. The following day, the misrouted rejection email arrived in the manufacturer's inbox. Assume the parties are in a jurisdiction that applies the mailbox rule to electronic communications. Was a contract formed? Answers: A. Yes, because the salon owner accepted the manufacturer's offer. B. Yes, because the mailbox rule applies. C. No, because the salon owner rejected the offer prior to accepting the offer. D. No, because the manufacturer received the salon owner's rejection.

Answer choice A is correct. The mailbox rule states that a timely sent acceptance is effective when sent, not upon receipt. However, if a communication is sent rejecting the offer, and a later communication is sent accepting the contract, the mailbox rule does not apply, and the first one to be received by the offeror prevails. Here, the salon owner rejected the offer, then sent an acceptance. Since the acceptance was received by the offeror first, the acceptance prevails. Answer choice B is incorrect because, as noted with respect to answer choice A, the mailbox rule does not apply when an acceptance is sent after a rejection. Answer choice C is incorrect because, while a rejection's effect is to terminate an offer, to be effective, the rejection must be communicated to the offeror. When a rejection is sent before an acceptance, the mailbox rule does not apply and the first communication received by the offeror prevails. Answer choice D is incorrect because a contract was formed when the manufacturer received the acceptance before the rejection. The fact that the manufacturer received the rejection the following day may affect the damages to which the manufacturer is entitled if the salon owner refuses to honor the contract, but it does not control the issue of the formation of the contract.

On March 1, the owner of a ferry boat that operated only during daylight hours during the summer months of June, July, and August entered into a written agreement with a man to serve as the captain of the boat for the upcoming season. On May 1, the owner contracted with a woman to serve as the captain of the boat. On May 30, the man was diagnosed with an illness, and the treatment for this illness prevented him from being employed until the following year. On May 31, the owner, learning of the man's illness, told him not to worry about their contract, as he had found someone else to serve as captain of the ferry boat. The woman served as captain of the ferry boat for the summer months of June, July, and August that year. On September 1, the man sued the owner for damages based on a breach of their contract. Will his suit succeed? Answers: A. No, because the man was unable to serve as the captain of the boat during the summer months. B. No, because the owner informed the man about the owner's contract with the woman prior to June 1 C. Yes, because the owner's contract with the woman constituted an anticipatory breach fo the owner's contract with the man. D. Yes, because the owner did not inform the man of the owner's contract with the woman until after the man learned he had an illness.

Answer choice A is correct. The owner's contract with the woman to serve as the captain of the ferry boat constituted an anticipatory breach of the owner's contract with the man, because the owner would not have been able to employ both the man and the woman as the captain of the ferry boat. Despite this breach, however, the owner's duty to pay damages to the man was discharged by the man's inability to have served as the captain of the ferry boat. Answer choice B is incorrect because, although the owner did tell the man about the owner's contract with the woman prior to the date on which the man was to begin serving as captain, this information constituted an anticipatory breach of the owner's contract with the man rather than a reason for excusing the owner from honoring that contract. Instead, it is the man's inability to serve as captain of the ferry boat during the summer months that discharges the owner's duty to pay damages. Answer choice C is incorrect because, although the owner's contract with the woman to serve as captain of the ferry boat did constitute an anticipatory breach of the owner's contract with the man, the owner's duty to pay damages to the man as a consequence of this breach was discharged by the inability of the man to have served as captain of the ferry boat during the summer months. Answer choice D is incorrect because, while the owner's delay in telling the man about the other contract was almost certainly not in good faith, there is no indication that this delay caused the man to forego other employment. Moreover, even had the man been able to secure alternative employment for the summer months, the man's illness would have prevented him from being employed during that time.

A wheat farmer contacted an agricultural services company in May to inquire about hiring workers for a five-day period toward the beginning of the summer-long harvest season to assist the farmer in harvesting his wheat crop. After some negotiations, the farmer entered into a written contract with the company to provide five workers for a five-day period starting in the first week of June for a cost of $5,000. On June 5, the company's workers went on strike. On June 9, the strike ended and the company's workers began harvesting wheat on the farmer's farm, and did so for five days. The farmer subsequently refused to pay the company, claiming the company's delay in performance excused his obligation to pay. Is the farmer's obligation to pay excused? Answers: A. No, because the contract did not contain a "time is of the essence" clause. B. Yes, because substantially performing does not excuse a breach for commercial contracts. C. Yes, because starting in the first week of June was an express condition of the contract. D. Yes, because the delay was a material breach as the harvesting season had already begun.

Answer choice A is correct. Under common law, a material breach of contract allows the nonbreaching party to withhold any promised performance and pursue remedies for the breach, including damages. A breach is considered minor when the breaching party has substantially performed. Here, both the parties agreed that time was not of the essence (because the completion date was not an express condition, as explained more fully below) and the delay does not appear to have been consequential (a somewhat nebulous start date combined with a summer-long harvesting season further supports this conclusion). For this reason, answer choice D is incorrect. Answer choice B is incorrect because it misstates the law, as there is no such rule that substantial performance does not apply to commercial contracts. Substantial performance does not apply to the sale of goods under the UCC, but does apply to contracts for services, such as the contract here. Answer choice C is incorrect because the "first week of June" start date was not an express condition. Express conditions generally contain language such as, "on condition that" or "provided that"

A caterer contracted with a local farmer for the delivery of three dozen fresh local eggs. The contract provided that because the caterer used the sterilized egg shells to serve one of her signature dessert recipes, the eggs needed to be a uniform color. The farmer delivered the caterer 20 white eggs and 16 speckled eggs. The caterer immediately e-mailed the farmer and informed him that she was rejecting the eggs because she could not use the inconsistent shells to serve her desserts. She also told him she could either sell them for him or save them for him to retrieve, and that she would wait for his instructions. The caterer saved the eggs for a week while she waited for the farmer's instructions, but he never responded. Finally, concerned that the eggs would spoil soon, she gave them to a friend. If the farmer brings a contract claim against the caterer for the contract price of the eggs she gave to her friend, will he succeed? Answers: A. No, because the caterer did not accept the eggs. B. No, because the caterer had no obligations regarding the nonconforming eggs. C. Yes, because the caterer had a duty to retain the eggs until the farmer retrieved them.

Answer choice A is correct. Under the UCC, if either the tender or the goods are nonconforming, then the buyer has the right to accept or reject all or part of the goods. Upon tender of nonconforming goods, a buyer must retain possession of the rejected goods for a reasonable period of time to allow the seller to reclaim them. However, a buyer's attempts in good faith to dispose of defective goods when the seller fails to give instructions within a reasonable time are not to be regarded as an acceptance. Here, the caterer properly rejected the eggs and tried to keep them until the farmer told her what to do with them, but she never received a response. Therefore, she did not accept the eggs, and the farmer cannot recover the contract price in a contract action. Answer choice B is incorrect because the caterer was obligated to notify the farmer that she was rejecting the eggs and to retain the eggs for a reasonable period of time for the farmer to reclaim them, which she properly did. Answer choice C is incorrect because the caterer was only required to retain the eggs for a reasonable amount of time. Based on the fact that she was concerned the eggs would spoil, she had already waited a reasonable amount of time with no response from the farmer when she gave the eggs to her friend. Answer choice D is incorrect. If rejected goods are perishable and the seller has no local agent to whom they can be returned, in the absence of other instructions from the seller, a merchant buyer is required to sell the goods on the seller's behalf. However, a non-merchant buyer is not required to sell the goods on the seller's behalf. Because the caterer is not in the business of selling eggs, she had no duty to sell these perishable goods on the farmer's behalf.

At the auction of construction equipment owned by a contractor, several lots were offered for bidding and the highest bids for each were accepted by the auctioneer. The auctioneer then announced that a lot that consisted of a backhoe was being auctioned off. Several bids for the backhoe were acknowledged by the auctioneer. Just before the auctioneer brought down her gavel, she glanced at the contractor. The contractor gave the auctioneer a prearranged signal. Acting in accord with the signal, the auctioneer stated that the backhoe was being removed from the auction. There had been no indication as to whether the auction was being held with or without reserve. The highest bidder on the backhoe, contending that he is now its owner, has brought suit against the contractor. How should the court rule? Answers: A. For the contractor, because the auctioneer has not brought down the gavel, announcing the completion of the sale of the backhoe. B. For the contractor, because the backhoe constituted equipment. C. For the highest bidder, because unless the seller reserves the right to withdraw an item from an auction, the seller may not do so once the auction has begun. D. For the highest bidder, because the seller forfeited his right to withdraw the backhoe by prearranging a signal with the auctioneer.

Answer choice A is correct. Unless specifically announced otherwise, an auction is with reserve, meaning that the seller has the right to withdraw an item from sale at any time before the auctioneer announces the completion of the sale. Here, because the backhoe was withdrawn from the sale before the auctioneer announced the completion of the sale by bringing down the gavel, the contractor remains the owner of the backhoe. Answer choice B is incorrect because the classification of the goods being auctioned is irrelevant to determining whether the goods can be withdrawn from the auction. Answer choice C is incorrect. As noted with respect to answer choice A, the default rule is that an auction is with reserve unless there is a specific announcement to the contrary. In addition, if goods in an auction are offered in lots, each lot represents a separate sale. Consequently, even though the auction had begun, the contractor retained the right to withdraw the backhoe, which was being auctioned as a separate lot, from the sale until the auctioneer announced the completion of its sale. Answer choice D is incorrect, as there is no prohibition on this type of behavior.

A woman emailed her friend, stating that she would like to buy the friend's teacup collection when "times aren't so tough. I would pay $1,000 for them." The friend responded with an email agreeing to the deal. The women did not exchange money or teacups and did not see each other until a year later. When they did see each other, the friend apologized for forgetting their discussion and said that she would deliver the teacups the next weekend and would accept a check at that time. The woman said she did not remember the discussion, but would pay $750 for the teacups. The friend responded, "Haven't we already discussed this? Sold." The next day, the friend turned the teacups over to the woman, who provided her with a check for $750. The friend immediately responded that she needed the check for the remaining $250. The woman kept the teacups. Is the woman liable for the remaining $250? Answers: A. No, because a contract had not been formed. B. No, because a contract was not formed until the day the women spoke in person C. Yes, because the original contract was for $250. D. Yes, because she kept the teacups.

Answer choice B is correct. A statement is an offer only if the person to whom it is communicated could reasonably interpret it as an offer. It must express the present intent of a person to be legally bound to a contract. The primary test of whether a communication is an offer is based on the objective theory of contracts; i.e., whether an individual receiving the communication would believe that he could enter into an enforceable deal by manifesting agreement to it. Here, the woman's email at the very most reflected a potential interest in buying the teacups at a future, unknown date. It does not reflect a present intent to be legally bound to buy the teacups. On the day of the oral discussion, however, she expressed a present intent to buy the teacups, and her friend would reasonably believe that they could enter into an enforceable deal on that day. Answer choice A is incorrect because on the day of the oral discussion, the women did form an agreement regarding the teacups. Answer choice C is incorrect because as indicated above, the original letter indicated merely a possible interest in buying the teacups at a later date but no present intent to be bound by such an agreement. Answer choice D is incorrect because she accepted the teacups based on the agreement entered into on the day of the oral discussion.

A homeowner entered into a contract with a landscaper. The contract specified that the homeowner would pay the landscaper $10,000 upon completion of a list of projects. The landscaper performed the work while the homeowner was away on vacation. When the landscaper sought payment, the homeowner refused, noting that a tree had not been trimmed as required by the contract. The landscaper responded that, since he would now have to forego other work in order to trim the tree, he would do it but only if the homeowner agreed to pay him a total of $10,500 for his services. The homeowner, desperate to have the work completed, agreed. Once the work was completed, however, the homeowner gave the landscaper a check for $10,000, and refused to pay more. The landscaper sued for breach of contract. Is the landscaper likely to succeed in his claim? Answers: A. No, because an enforceable contract cannot be renegotiated. B. No, because there was no consideration for the promise to pay $10,500 and no unanticipated circumstances arose. C. Yes, because there was a valid modification of the contract. D. Yes, because the landscaper suffered a detriment by foregoing other work.

Answer choice B is correct. At common law, a promise to perform a preexisting legal duty does not qualify as consideration because the promisee is already bound to perform. In this case, the landscaper had a preexisting legal duty to trim the tree, and thus there was no consideration to support the homeowner's promise to pay an additional $500. Answer choice A is incorrect because an enforceable contract may be renegotiated. Even when there is a preexisting legal duty, there will be consideration if the promisee gives something in addition to what is already owed or varies the preexisting duty. Answer choice C is incorrect because modification of a services contract must be supported by consideration. Or some circumstances that were not anticipated when the contract was made must have arisen, and modification is fair and equitable in light of those circumstances. Answer choice D is incorrect because the fact that the landscaper had to forego other work would not serve as consideration in this case because the landscaper was under a preexisting legal duty.

A supplier contends that a dentist called and ordered 100 boxes of disposable paper masks, which the supplier's representative immediately accepted. The supplier also promptly sent the dentist a signed, written acknowledgment of the alleged order, which reflected the supplier as seller and the dentist as buyer, as well as the number and type of masks ordered. However, the writing did not indicate the price of the masks. The dentist admits to receiving the acknowledgment the following day, but asserts that she took no action regarding it. Two weeks later, the dentist received a shipment of 100 boxes of masks from the supplier along with an invoice that reflected the price of $9.99 per box. The dentist immediately called the supplier and asserted that she had never ordered the masks. Will the Statute of Frauds prevent the supplier from enforcing this contract against the dentist? Answers: A. No, because an oral contract made between merchants is enforceable without a writing. B. No, because the dentist did not respond to the written acknowledgment in a timely manner. C. Yes, because the acknowledgment did not indicate the price of the masks. D. Yes, because the price of the masks exceeds the threshold for the application of the Statute of Frauds to a sale of goods.

Answer choice B is correct. Generally, a contract that falls within the Statute of Frauds is unenforceable unless evidenced by a writing. The writing must (i) be signed by the party to be charged and (ii) contain the essential elements of the deal. A contract for the sale of goods for a price of at least $500 falls with the Statute of Frauds. Consequently, the order of 100 boxes of masks at $9.99 per box is subject to the Statute of Frauds. Although the acknowledgment sent by the supplier to the dentist was otherwise sufficient to satisfy the requirements of the Statute of Frauds, it was not signed by the dentist. However, if both parties are merchants and a memorandum sufficient against one party is sent to the other party, who has reason to know its contents, and the receiving party does not object in writing within 10 days, then the contract is enforceable against the receiving party even though she has not signed it. Here, this merchant's exception applies to the dentist who received the acknowledgment of her order and did not reply for more than 10 days after receiving it. Answer choice A is incorrect. Although a memorandum of an oral agreement between merchants may permit the enforcement of the agreement against another merchant, there is no blanket exception to the Statute of Frauds that permits enforcement of an oral agreement between two merchants. Therefore, this answer choice is a misstatement of the law. Answer choice C is incorrect. Under the Statute of Frauds for the sale of goods, the price of the goods need not be included in the contract. If the price is omitted, the parties can present evidence of the agreed-upon price or (if that fails) the market price. Therefore, the failure of the acknowledgment to reflect the price of the masks does not prevent the acknowledgment from satisfying the merchant's exception to the Statute of Frauds. Answer choice D is incorrect. The Statute of Frauds for a sale of goods does apply to this transaction since the contract involves the sale of goods at a total price of $999 (100 x $9.99) and the threshold for application of the UCC Statute of Frauds is the sale of goods at a price of at least $500. Since the dentist did not sign the acknowledgment, it is true that generally the acknowledgment cannot serve as evidence of the existence of a contract. However, the acknowledgment sent by the supplier does satisfy the memorandum exception to the Statute of Frauds that is applicable when both parties are merchants.

A shoe manufacturer contends that the owner of a shoe store called and ordered 50 pairs of Oxford-style dress shoes at $100 per pair to be shipped within three weeks and that the manufacturer's representative immediately accepted this order. The manufacturer promptly sent the owner a signed, written acknowledgment of the alleged order that reflected the manufacturer as seller and the shoe store owner as buyer, as well as the number and style of shoes, but that did not indicate the price of the shoes. The owner admits to receiving the acknowledgment the following day and taking no action regarding it. Two weeks later, the owner received a shipment of 50 pairs of Oxford-style dress shoes. The owner immediately called the manufacturer and asserted that he had never ordered the shoes. Will the Statute of Frauds prevent the manufacturer from enforcing this contract against the owner? Answers: A. No, because an oral contract made between merchants is enforceable. B. No, because the owner received and did not respond to the written acknowledgment in a timely manner. C. Yes, because the acknowledgment did not indicate the price of the shoes. D. Yes, because the price of the shoes exceeds the $500 threshold of the Statute of Frauds.

Answer choice B is correct. Generally, a contract that falls within the Statute of Frauds is unenforceable unless evidenced by a writing. The writing must (i) be signed by the party to be charged and (ii) contain the essential elements of the deal. A contract for the sale of goods for a price of at least $500 falls with the Statute of Frauds. Consequently, the purported order of 50 pairs of shoes at $100 per pair is subject to the Statute of Frauds. Although the acknowledgment sent by the manufacturer to the owner is otherwise sufficient to satisfy the Statute of Frauds, it was not signed by the owner. However, if both parties are merchants and a memorandum sufficient against one party is sent to the other party, who has reason to know its contents, and the receiving party does not object in writing within 10 days, then the contract is enforceable against the receiving party even though he has not signed it. Here, this merchant's exception applies to the owner who received the acknowledgment of his order and did not reply for more than 10 days after receiving it. Answer choice A is incorrect. Although a memorandum of an oral agreement sent by one merchant may permit the enforcement of the agreement against another merchant, there is no blanket exception to the Statute of Frauds that permits enforcement of an oral agreement between two merchants. Answer choice C is incorrect. Under the Statute of Frauds for the sale of goods, the price of the goods need not be included in the contract. If the price is omitted, the parties can present evidence of the agreed-upon price or (if that fails) the market price. Therefore, the failure of the acknowledgment to reflect the price of the shoes does not prevent the acknowledgment from satisfying the merchant's exception to the Statute of Frauds. Answer choice D is incorrect. Although the Statute of Frauds for a sale of goods does apply to this purported transaction and was not satisfied because the owner did not sign the acknowledgment, the acknowledgment sent by the manufacturer satisfies an exception to the Statute of Frauds when both parties to the oral agreement are merchants.

A hotelier owned a hotel and the surrounding land. A portion of the land immediately behind the hotel was unused, and the hotelier wanted to develop it and install an in-ground pool. She contacted a general contractor in January, and the parties agreed that the contractor would build out a large terrace complete with a fifty-foot in-ground pool. They agreed to a price $400,000, with the contract to be completed before March 15, payable upon completion of the work. The hotelier stressed to the contractor the importance of completing the work before March 15 because that was when her vacation season began and she planned a special advertisement of the pool terrace to boost sales. The hotelier paid the contractor the $150,000 initial payment, but the contractor abandoned the job in late January without performing any work on the site. The hotelier hired a second contractor in February, who charged her $550,000 to complete the job. Despite the rush, the terrace was not ready until April 15, and the hotelier lost $70,000 in net profit due to documented reservation cancellations because of the unfinished construction. What is the most a court could properly award the hotelier? Answers: A. $550,000 B. $370,000 C. $220,000 D. $150,000

Answer choice B is correct. In construction contracts, the general measure of expectation damages for failing to perform is the difference between the contract price and the cost of completion by another builder. Here, that difference is the $550,000 cost to hire the replacement contractor minus the $400,000 contract price, or $150,000. Further, consequential damages may be awarded for reasonably foreseeable losses to a nonbreaching party that go beyond expectation damages, such as loss of profits. Damages are recoverable if they were the natural and probable consequence of breach, or if they were in the contemplation of the parties at the time the contract was made, and may be calculated with reasonable certainty. Here, lost profits due to room cancellations is a natural and probable consequence of ongoing construction at the hotel because guests may have specifically wanted to stay in a hotel with a pool or else avoided a hotel that had ongoing construction on the premises. Further, the hotelier had specifically told the contractor that she planned to specifically advertise the pool terrace to boost sales. Additionally, the hotelier had detailed documentation to support her claim of lost profits. Therefore, a court could award her the $70,000 in lost profits. Finally, the hotelier would be able to recover the $150,000 that she originally paid to the first contractor as a deposit. Therefore, the correct measure of damages would be the $70,000 in lost profits, (consequential damages) plus the $150,000 in expectation damages, plus the $150,000 deposit, for a total of $370,000. Answer choice A is incorrect because this assumes the hotelier would receive the entire $550,000 replacement contract price (an incorrect measure of expectation damages). It also ignores the $70,000 in consequential damages and the deposit. Answer choice C is incorrect because it correctly measures expectation damages and consequential damages, but fails to take into account the deposit. Answer choice D is incorrect because it incorrectly omits the $70,000 in consequential damages and the deposit that the court could award.

A woman sent an offer to sell her office printer to her friend for $450. In her offer, the woman mentioned that an acceptance could be mailed to her business address, and that the friend should let her know within the next couple of weeks whether she was interested. The friend needed an office printer, so she immediately accepted the woman's offer by mailing a letter to the woman's home address. The woman only checked her mailbox at home once a week because she received so much junk mail, so she did not see the acceptance letter. Thinking that her friend was not interested, the woman sold her office printer to a different person. The next day, the friend came to the woman's house with a check for $450. The woman told the friend she had already sold the office printer. Will the friend succeed in an action for breach of contract? Answers: A. Yes, because the offer was irrevocable for at least two weeks. B. Yes, because the woman did not specify that mailing an acceptance to her business address was the only mode of acceptance. C. No, because it was not reasonable for the friend to mail her acceptance to the woman at the woman's home address. D. No, because the woman did not see the friend's acceptance letter.

Answer choice B is correct. The offeror can dictate the manner and means by which an offer may be accepted. Unless the offeror specifically requires the offeree to accept in a particular manner or by using a particular means, the offeree can accept in any reasonable manner and by any reasonable means. Here, the woman did not specify that the acceptance could only be mailed to her business address; she merely suggested that an acceptance could be mailed to her business address. Thus, another reasonable mode of acceptance, such as mailing it to the woman's home address, is permissible. Accordingly, answer choice C is incorrect. Answer choice A is incorrect because offerors may generally revoke offers at any time prior to acceptance, even if the offeror claims that the offer will be held open. Here the friend would not succeed because the offer was irrevocable, but because she sent her acceptance before the offer was revoked. Answer choice D is incorrect. Under the mailbox rule, an acceptance is valid open posting, not receipt. Here, the friend's acceptance was effective when she mailed the letter to the woman's home. Upon mailing the letter, an enforceable contract was created between the woman and the friend, even though the woman did not see the friend's acceptance letter.

A jeweler and a goldsmith signed a written agreement to provide as follows: "For $3,000, the goldsmith shall sell to the jeweler a size six gold ring setting that the jeweler shall select from only the goldsmith's white gold ring designs." The agreement did not address any other specific terms with regard to the business arrangement between the jeweler and the goldsmith. When the jeweler arrived to select a ring, he refused to select one, claiming that the goldsmith, immediately prior to the execution of the written agreement, had orally agreed to broaden his choices to also include rose gold ring designs. The jeweler claimed that the goldsmith had, at the same time, also orally agreed to include a set of earring settings, valued at $1,000, as an incentive for his continued business. The goldsmith refused to sell to the jeweler any of his rose gold ring designs or include the earring settings. If the jeweler sues the goldsmith for damages, how should the court handle the evidence of the alleged oral agreements? Answers: A. The court should admit the evidence as to both the earring settings and the option to choose a rose gold ring design. B. The court should admit the evidence as to the earring settings but not the option to choose a rose gold ring design. C. The court should admit the evidence as to the option to choose a rose gold ring design but not the promise to include the earring settings. D. The court should admit neither the option to choose a rose gold ring design nor the promise to include the earring settings.

Answer choice B is correct. The parol evidence rule generally prevents a party to a written contract from presenting prior extrinsic evidence that contradicts the terms of the contract as written. If the writing completely expresses all of the terms of the parties' agreement, then it is a total integration, and the parties cannot introduce any extrinsic evidence (oral or written) of prior or contemporaneous understandings or negotiations. If, on the other hand, the writing sets forth the parties' agreement about some terms, but not all terms, then it is a partial integration. The UCC essentially presumes that a contract is a partial integration. Here, the UCC applies, so the contract will be presumed to be a partial integration, allowing additional consistent terms unless they certainly would have been included. The agreement between the jeweler and the goldsmith set forth only the size, price, and type of gold for the gold ring setting; it did not set forth any other relevant terms to their business agreement. When a writing is a partial integration, the parties are permitted to introduce supplementary extrinsic evidence (oral or written) of other terms as long as the evidence is consistent with the writing, but not if the evidence contradicts the terms of the writing. Here, the evidence about the earring settings only supplements the written agreement. However, the evidence about the option to choose a rose gold ring design directly contradicts the written agreement, which explicitly states that the jeweler can choose only from the white gold ring designs. Therefore, only the evidence as to the earring settings should be admitted. Answer choices A and C are incorrect, as the evidence about the option to choose a rose gold ring design expressly contradicts the written agreement and thus cannot be admitted. Answer choice D is incorrect, as the evidence about the earring settings only supplements the written agreement and thus would be admissible under the parol evidence rule.

A dancer signed a contract with a traveling circus to travel and perform as an aerialist for six months. The contract provided that the dancer would be paid $500 per week and would be guaranteed employment for the full six months, with an option to renew the contract for the next traveling season. Excited for the opportunity to perform for a traveling circus, the dancer turned down an invitation to dance with a theatre group for the same time period as the circus contract. The dancer brought an action against the circus for breach of contract. If the danger wants to recover the highest possible amount of damages, which of the following is the dancer's best legal theory? Answers: A. The dancing contract with the circus is legally severable into weekly units. B. The dancer's failure to perform for two weeks was not a material breach of the contract. C. The dancer's performance of the terms of the contract is impracticable given her injury. D. The dancer detrimentally relied on the contract by declining the other dancing job.

Answer choice B is correct. Under common law, a material breach of contract occurs when the nonbreaching party does not receive the substantial benefit of its bargain. A material breach of a contract allows the nonbreaching party to withhold any promised performance and pursue remedies for the breach, including damages. A breach is considered minor when the breaching party has substantially performed. In this case, if the dancer is able to successfully argue that a two-week delay is only a minor breach and that she has otherwise substantially performed under the contract, then she will be able to recover damages. Answer choice A is incorrect because a severable contract would work against the dancer's desired recovery. A divisible or installment contract is one in which the various units of performance are divisible into distinct parts. Recovery is limited to the amount promised for the segment of the contract performed. Thus, if the contract was severable into weekly units, then the dancer would not have a strong claim to enforce the remainder of the contract, and she would be able to recover only for the two weeks that she did perform for the circus before injuring herself. Answer choice C is incorrect. A party's duty to perform can be dismissed by impracticability. In a personal-services contract, as is the case here, performance is considered impracticable if the performing party to the contract dies or becomes incapacitated. Here, if a court found that the dancer's performance under the contract was impracticable due to her injury, then it would result in a discharge of the contract, which would work against the dancer's desired recovery. Answer choice D is incorrect. Promissory estoppel is referred to as a "consideration substitute." The doctrine of promissory estoppel (detrimental reliance) can be used under certain circumstances to enforce a promise that is not supported by consideration. This doctrine would not help the dancer because there was an enforceable contract here that was supported by consideration. Estoppel is a consideration substitute, and this would not help the dancer recover damages.

A wholesaler of bicycle chains sent a retailer the following fax on December 1: "Because of your continued loyalty as a customer, I am prepared to sell you up to 1,000 units of Bicycle Chain Model D at $7.50 per unit, a 25% discount off our original $10.00 price. This offer will remain open for 7 days." The fax lacked a full, handwritten signature, but was on the wholesaler's letterhead and had been initialed by the wholesaler's head of sales. On December 4, the wholesaler's head of sales called the retailer and informed the retailer that he had decided to revoke his December 1 offer. On December 5, the retailer placed an order for 1,000 bicycle chains, stating that he would pay the discounted price of $7.50 per unit. What is the correct value of the order placed by the retailer? A. $7,500, because the wholesaler's revocation was not in writing. B. $7,5000, because the wholesaler was bound to keep the offer open for 7 days. C. $10,000, because the offer was not signed by the wholesaler. D. $10,000, because the retailer did not provide consideration to hold the offer open.

Answer choice B is correct. Under the UCC's firm offer rule, an offer to buy or sell goods is irrevocable if the offeror is a merchant, there is an assurance that the offer is to remain open, and the assurance is contained in a signed writing from the offeror. No consideration by the offeree is needed to keep the offer open. Here, all three conditions are satisfied (note that letterhead and an agent's initials suffices as a "signature"), and the wholesaler's offer was irrevocable until the offer period expired. The retailer's order was within the 7-day window, and thus the order for 1,000 units is priced at $7.50 each, or $7,500 total. Answer choice A is incorrect because the wholesaler was bound by the UCC firm offer rule, and his offer was irrevocable regardless of the method of his attempted revocation. Answer choice C is incorrect because the initials of the head of sales on the letterhead are sufficient to constitute a signed writing under the UCC. Answer choice D is incorrect because, under the UCC's firm offer rule, no consideration was required to hold the offer open.

An owner of a rare eighteenth-century chest offered to sell it to a connoisseur of antiques for $75,000. The connoisseur countered that she would buy the chest for $50,000. The owner rejected this price. The owner and connoisseur then executed a written agreement for the sale of the chest at a price to be determined only by an antiques dealer whose expertise in valuing this rare item they both trusted. The dealer examined the chest. He told the owner and the connoisseur that he had to do further research on the chest, but that he would let them know his decision in several days. Unfortunately, the dealer died before doing so. A reasonable price can be established for the chest by court. Is there likely to be an enforceable contract? Answers: A. No, because the price of the chest was not determined at the time the agreement was executed. B. No, because the owner and the connoisseur did not intend to be bound unless the dealer set the price of the chest. C. Yes, because a reasonable price can be established for the chest by the court. D. Yes, because the owner and the connoisseur executed a written agreement for the sale of the chest.

Answer choice B is correct. When parties enter into an agreement for the sale of goods and the price has not been set, there is no contract if the agreement reflects an intent not to be bound unless the price is subsequently set and the price is never set. Here, the owner and the connoisseur agreed to the sale of the chest only at a price set by the dealer. Because the dealer did not set the price before his death, there is no enforceable agreement for the sale of the chest. Answer choice A is incorrect because such an agreement may be enforceable despite an unresolved price at the time the agreement is entered into, as long as the price is arrived at in the manner agreed to by the parties. Here, had the dealer named a price for the chest, there would have been a contract that could have been enforced by either party against the other. Answer choice C is incorrect. Although a court may fix a reasonable price for goods when the parties to a contract fail to do so, the parties must intend to enter into a contract for the goods despite the absence of the price term. Here, the owner and the connoisseur did not ignore the price term, but rather set out a specific and exclusive method by which the price was to be determined. Because, due to the death of the dealer, that method cannot be used to determine the price, and the parties did not provide for an alternative method, the implication is that the parties did not intend to be bound if that specified method failed. Answer choice D is incorrect. Although the owner and the connoisseur did execute a written agreement, this agreement left the price of the chest to be determined by the dealer. Because the dealer's death makes it impossible for him to do so, the agreement fails. The parties' disagreement over the price of the chest prior to entering into the agreement also suggests that the parties did not have the intent to be bound if the dealer were unable to determine a price.

The owner of a fur coat stored it with the furrier from whom she bought the coat during the warm months of the year. While the coat was at the furrier, a salesperson, mistakenly thinking that the coat was for sale, sold it to a customer. The customer was allowed to reduce the purchase price by the amount of an outstanding debt owed by owner of the furrier to the customer; the customer paid the remainder in cash. In the process of purchasing the coat, the customer was told by the salesperson about the furrier's storage service, but, like the salesperson, was unaware that the coat was not part of the store's merchandise. After the sale, the owner learned of the transaction between the furrier and the customer. Since the coat had significant sentimental value to the owner, the owner sought its return from the customer. When the customer refused, the owner filed an action to recover the coat from the customer. Will the owner prevail? Answers: A. Yes, because the furrier transferred only voidable title in the coat to the customer. B. Yes, because the customer did not give full value in acquiring the goat. C. No, because the customer was a good faith purchaser of the coat that had been entrusted to the furrier. D. No, because the owner is entitled to damages from the furrier.

Answer choice C is correct. A good faith purchaser of goods in the ordinary course of business from a merchant takes good title to the goods if the goods have been entrusted by the owner to the merchant and the merchant deals in teh same kind of goods. Here, the customer bought hte fur coat at a furrier where it had been left by the owner. The customer was unaware that the coat belonged to the owner. Answer choice A is incorrect because although the furrier did not have title to the goods, the furrier could nevertheless transfer good title. Answer choice B is incorrect because the purchaser did give value: satisfaction of a pre-existing debt constitutes value. Answer choice D is incorrect because the owner's right ot seek damages from the furrier does not, in itself, preclude the owner from seeking to recover the coat from the customer. The entrustment of the coat to a furrier and the subsequent sale of that good to a good faith purchaser, however, does.

A licensing agreement provided that a manufacturer could use an inventor's patent in manufacturing its products for ten years. Immediately thereafter, the inventor assigned his rights to receive payments pursuant to the licensing agreement to a corporation in which he was the controlling shareholder. The inventor did not receive compensation for this assignment. The inventor, upon his death five years later, devised his stock in the corporation to his daughter, and all of his remaining property to his son. To whom should the manufacturer make its payments under the licensing agreement? Answers: A. The corporation B. The inventor's daughter C. The inventor's son D. The manufacturer's obligation to make payments under the licensing agreement terminated upon the death of the inventor.

Answer choice C is correct. A gratuitous assignment of contract rights automatically terminates upon the death of the assignor. Accordingly, the assignment to the corporation was revoked upon the inventor's death. However, a contract generally does not terminate upon the death of one of the parties. Thus, the manufacturer remains obligated to make its payments under the licensing agreement to a person who, by the terms of the inventor's will, has been devised the right to receive those payment. Because the right to the inventor's property, other than his corporate stock, was devised to his son, his son is entitled to receive the licensing payments from the manufacturer for the remaining term of the licensing agreement. Answer choice A is incorrect because the corporation's right to receive the payments pursuant to the licensing agreement terminated upon the inventor's death. Answer choice B is incorrect. Although the inventor's daughter is entitled to his corporate stock, the inventor's property, including the right to receive payments under the licensing agreement as well as the patent itself, was devised to the inventor's son. Answer choice D is incorrect because the inventor's death does not terminate the manufacturer's right to continue to use the inventor's patent for another five years and similarly does not terminate the manufacturer's obligation to make payments for that use pursuant to the licensing agreement for that time period as well.

Under the terms of an insurance contract, a casualty insurance company required an insured to give the company written notification of a casualty within 30 days of its occurrence. A wind storm caused a large tree to topple onto the insured's dwelling. The insured, who was away on vacation at the time of the storm, did not discover the casualty until she returned three weeks after the casualty. That same day, the insured contacted the company by phone and notified the company of the casualty. Three days later, the company sent an agent to the dwelling. The agent, with actual authority on behalf of the insurance company, told the insured that she did not have to take any further action with regard to notifying the company of the claim. A week after this conversation, the insured, having taken no action in reliance on the agent's statement, received a written communication from the insurance company, indicating that the company required the insured to fill out the enclosed notification form and return it to the company within 30 days in order for the company to cover the insured's loss. The insured failed to fill out the form for 45 days and the insurance company refused to pay for the casualty loss. The insured sued the insurance company, and the company defended against the action on the ground that the insured failed to give written notice of the casualty to company. How should the court rule? Answers: A. For the insurance company, because the company reinstated the written notice condition and gave the insured reasonable time to comply. B. For the insurance company, because the insured did not detrimentally rely on the company's waiver before the company reinstated the written notice condition. C. For the insured, because the company's agent waived the written notice condition and the company failed to timely reinstate it. D. For the insured, because, once waived, the company could not reinstate the written notice condition.

Answer choice C is correct. A party to a contract whose duty is subject to a condition can waive the condition, either by words or by conduct. In addition, that party may reinstate the condition if (i) the waiving party communicates a retraction of the waiver before the condition is due to occur, and (ii) the other party has not already suffered detrimental reliance. Here, although the company's agent, acting on behalf of the insurance company, could and did waive the written notice condition, the company's attempt to reinstate this condition was not timely, as it occurred after the condition, the initial 30-day period for providing written notification, had lapsed. Answer choice A is incorrect. Although the insurance company could have reinstated the waived condition, and the provision of an additional 30 days in which to give the insurance company written notice was most likely a reasonable time, the company did not act to reinstate the condition before the condition expired. Answer choice B is incorrect because, even though the insured did not detrimentally rely on the company's waiver, the insurance company could not reinstate the waived condition after the condition expired. Answer choice D is incorrect because, even though the insurance company did waive the written notice condition, the company could have reinstated the condition, as the insured had not relied on the waiver, had the company acted before the condition was due to occur.

On January 5, a buyer and seller contracted for the delivery of 100 widgets if they could be delivered by February 20. The agreement was made in a writing signed by both parties and provided that the buyer would pay the contract price of $1,000 upon delivery. On February 3, the buyer and the seller orally agreed to postpone delivery until March 1. However, when the widgets arrived on March 1, the buyer refused to accept or pay for the widgets. If the seller sues the buyer for breach of contract, who is most likely to succeed in the action? Answers: A. The buyer, because any modification of the parties' contract must satisfy the Statute of Frauds. B. The buyer, because the agreement on February 3 was not supported by consideration. C. The seller, because the oral agreement on February 3 waived the February 20 delivery date. D. The seller, because the contract modification on February 3 was immediately binding on both parties.

Answer choice C is correct. A party whose duty is subject to the condition can waive the condition, either by words or by conduct. A party who indicates that a condition will not be enforced may be estopped from using that condition as a defense if the other party reasonably relied on the party's words or conduct that the condition had been waived. Here, the buyer and seller agreed to postpone delivery (i.e., the buyer waived the condition that the widgets be delivered by February 20). The seller relied on that waiver (as evidenced by the fact that he did not deliver the widgets until March 1) and it would be unjust for the buyer to now claim that the contract was breached. Therefore, the seller will prevail because the February 20 delivery date was waived. Answer choice A is incorrect. A promise that the promisor should reasonably expect to induce action or forbearance on the part of the promisee or a third person and that does indue the action or forbearance is enforceable notwithstanding the Statute of Frauds if injustice can be avoided only be enforcement of the promise. Here, the buyer's indication that delivery would be accepted on March 1 prevents the buyer form using the seller's failure to deliver by the February 20 delivery deadline to justify the buyer's failure to perform under the contract. Answer choice B is incorrect because consideration is not needed to modify a contract under the UCC. Answer choice D is incorrect because contract modification (i.e., the waiver) was not technically enforceable until there had been a detrimental reliance. The requirements of the Statute of Frauds must be satisfied if the contract as modified is within its provisions and as a contract for the sale of goods over $500, such a modification would require a writing. However, once the buyer let the original delivery date pass, the buyer was estopped from using the passing of that deadline as justification for failing to perform.

At the beginning of an auction, which consists of 25 lots of goods owned by the same person, the auctioneer announced that the right of the seller to withdraw goods was not reserved. After the sales of two lots were completed but prior to the auctioneer's call for bids on the third lot, the owner sought to withdraw the third lot from the auction. Can the owner properly withdraw the third lot? Answers: A. No, because the auctioneer announced that the auction was without reserve. B. No, because sales of other lots owned by the same person had been completed. C. Yes, because the auctioneer has not called for bids on the third lot. D. Yes, because the auctioneer has not announced the completion of the sale of the third lot.

Answer choice C is correct. In an auction, each lot is considered to be a separate sale. In a no-reserve auction, after the auctioneer calls for bids on the goods in a lot, the goods cannot be withdrawn unless no bid is received within a reasonable time. With regard to the third lot, because the auctioneer had not called for bigs on that lot, the goods can be withdrawn. Answer choice A is incorrect. Even though the right of the seller to withdraw goods is not reserved, a seller possesses the right to withdraw the goods until the auctioneer has called for bids on the goods. Consequently, the fact that other lots owned by the same person have been sold in the same auction does not affect the right of the owner of a lot to withdraw that lot from the auction. Answer choice D is incorrect because it states the rule for withdrawal of goods from an auction were the auction is with reserve.

On March 1, a company contracted with a singer for the singer to perform for the company picnic on May 1 for a fee of $10,000. On March 17, the singer informed the company that she signed a contract to film a movie. She suggested that the company hire another signer to take her place at the picnic. On April 1, the company hired the recommended replacement singer to perform at its picnic for $15,000. On April 25, the original singer informed the company that she had decided not to take the movie deal, and will be available to perform on May 1. Even though the original singer arrived at the picnic on May 1 ready to sing, the company let the replacement singer perform. The company refused to pay $10,000 to the original singer. Is the company likely to prevail in a breach of contract claim against the original singer? Answers: A. No, because the company prevented the original singer from fulfilling her contractual obligation by refusing to let her perform on May 1. B. No, because the original singer retracted her repudiation before the scheduled performance. C. Yes, because the company hired the replacement singer as a substitute for the original singer before she retracted her repudiation. D. Yes, because the replacement singer's consent to the delegation of the original singer's duties did not create a novation.

Answer choice C is correct. The company's contract with the replacement singer as a substitute for the original singer's services constituted a material change in position and thus terminated the original singer's ability to retract her repudation. Answer choice A is incorrect because acting in reasonable reliance on an unequivocal repudiation does not violate the doctrine of prevention. Answer choice B is incorrect because the company's material change in position happened before the original singer tried to retract her repudiation. Answer choice D is incorrect because even if this personal service contract could be delegated, the original singer's suggestion that the company hire the second singer does not constitute an effective delegation of her contractual duties.

A maker of hand-woven rugs contracted with a supplier to provide yarn made from sheep's wool. The written contract specified that, for four years, the supplier would provide the rug maker with 2,000 spools of yarn made from 100% sheep's wool per month, at $10 per spool, for a total of $20,000. Two years into the contract, the supplier sent 2,000 spools of yarn to the rug maker made from 90% sheep's wool and 10% synthetic fiber. The rug maker sent a check to the supplier for $15,000 for the shipment, and added a clear note on the check stating that the payment was in full for the shipment, but was $5,000 less due to the synthetic fiber in the yarn. The supplier promptly deposited the check, and then four months later filed suit against the rug maker for the remaining $5,000. The supplier has submitted evidence of the written contract, and the rug maker has submitted evidence of the deposited check. What is the rug maker's best defense in this situation? Answers: A. The rug maker's and supplier's good faith dispute over the yarn composition suspended the rug maker's obligation to pay the remaining $5,000. B. The act of knowingly depositing the check for $15,000 by the supplier was a novation that relieved the rug maker from any further liability. C. The supplier deposited the check for $5,000 less than the contract price, thereby discharging the rug maker of any further duty to pay the remaining amount for that month's shipment. D. By depositing the check, the supplier was estopped from claiming that the rug maker owed him an additional $5,000.

Answer choice C is correct. This is considered an "accord and satisfaction," which discharges both the original contract and the accord contract. Under an accord agreement, a party to a contract agrees to accept a performance from the other party that differs from the performance that was promised in the existing contract, in satisfaction of the other party's existing duty. Generally, consideration is required for an accord to be valid. By compromising, each party surrenders its respective claim as to how much is owed. If a claim is subject to dispute, it can be discharged if the person against whom the claim is asserted in good faith tenders a negotiable instrument (e.g., a check) that (i) is accompanied by a conspicuous statement indicating that the instrument was tendered as full satisfaction of the claim (e.g., "payment in full"), and (ii) the claimant obtains payment of the instrument. Here, the rug maker compromised by accepting the 90% wool yarn, and the supplier compromised by accepting $15,000 rather than $20,000. Thus, there was an accord and satisfaction and the rug maker is not liable for the remaining $5,000. Answer choice A is incorrect, because the good-faith dispute did not suspend the duty of the rug maker to pay for the yarn supplied. In the context of an accord and satisfaction, it is only the existence of an accord agreement that suspends the original duty of a party. Answer choice B is incorrect, as this is not a novation. A novation is the substitution of a new contract for an old one when the original obligor is released from his promises under the original agreement and a new obligor becomes liable. Here, there are only two original parties-the rug maker and the supplier. Answer choice D is incorrect because estoppel requires not only an assertion by a party but also justifiable reliance on the assertion by the party to whom the assertion is made, to that party's detriment. Here, even assuming that the supplier's deposit of the check constituted an assertion, there is no evidence that the rug maker has relied on this assertion to his detriment.

Seeing his elderly neighbor struggle to shovel large amounts of snow from a sidewalk, a man decided to shovel the neighbor's sidewalk himself. The man wanted to help the neighbor so she would not injure herself. He figured that if anyone else used the sidewalk, he could charge them later for the work performed. The man continued to shovel the sidewalk for the remainder of the winter. The neighbor knew the man was doing the shoveling and never objected. She even wrote him a note on the first day of spring indicating that she would love to pay him for the service. A second neighbor regularly used the particular sidewalk as well. When the man ran into the second neighbor at the grocery store one day, the second neighbor said he would pay the man for taking such good care of the neighborhood. At the end of the winter, the man placed a bill on the doorstep of both the elderly neighbor and the second neighbor. The service charges were comparable to the reasonable market rate for such services. The jurisdiction applies the material benefit rule. From whom can the man can likely obtain payment? Answers: A. Neither the elderly neighbor nor the second neighbor B. The elderly neighbor only C. Correct Answer: The second neighbor only D. The elderly neighbor and the second neighbor

Answer choice C is correct. Under the material benefit rule, when a party performs an unrequested service for another party that constitutes a material benefit, the performing party (here, the man) can enforce a promise of payment made by a party who benefits from the service rendered. In this case, both neighbors benefited from the shoveling and both offered payment. However, the material benefit rule is not enforced when the performing party rendered the services without the expectation of compensation. Here, the facts indicate that the man wished to shovel the snow for his elderly neighbor in a gift capacity—i.e., so she would not injure herself. He had no such motivation for the second neighbor and can enforce that payment. Answer choice A is incorrect because the man can enforce payment against the second neighbor, as he did not intend for that neighbor to benefit from the gift. Answer choice B is incorrect. The man cannot enforce payment against the elderly neighbor even though she benefited from his services because he intended the service to be a gift. Answer choice D is incorrect because the man cannot enforce payment against the elderly neighbor.

A grocery store whose main customer base was families with young children contacted a cereal manufacturer. After various discussions regarding the cereal and the box, the two parties entered into a written contract whereby the grocery chain would purchase 10,000 boxes of children's cereal on a monthly basis for $5,000, due upon delivery. The contract further stated that the cereal would be shaped like donuts, and each piece of cereal would be one of the seven colors of the rainbow. Finally, in listing the primary ingredients, the contract stated that the cereal would contain high fructose corn syrup. When the first shipment arrived, the grocery chain refused to pay the $5,000 and repudiated the contract. The cereal manufacturer sued the grocery store for damages, and admitted the contract between the two parties into evidence. The grocery chain then attempted to offer evidence regarding the discussions that occurred between the two parties prior to the execution of the contract. It claimed that the cereal manufacture had orally agreed to use evaporated cane juice as a sweetener, not high fructose corn syrup, and that the cereal manufacturer would also include a small prize in each cereal box at a cost of a penny a box. In deciding whether to admit evidence of the oral agreement, the court will most likely Answers: A. Admit neither piece of evidence from the oral agreement B. Admit only the evidence regarding the use of evaporated cane juice as a sweetener C. Admit only the evidence regarding the small prize in each cereal box. D. Admit both pieces of evidence from the oral agreement.

Answer choice C is correct. Under the parol evidence rule, evidence that contradicts the writing is generally inadmissible, but evidence that supplements a contract that is partially integrated is admissible if it is consistent with the writing and does not contradict its terms. The UCC essentially presumes that a written contract is only a partial integration. Evidence of any outside term is admissible unless a court concludes that the parties "certainly" would have included the term in the written contract. Here, the evidence about the small prizes supplements the written agreement and does not contradict it. The cost of the prize, both in actual amount (a penny) and in relationship to the overall price of the cereal (1 cent v. 50 cents ($5,000/10,000)) suggests that the parties would not certainly have included this term in their contract. Accordingly, this evidence is not admissible. However, the evidence about using evaporated cane juice as a sweetener instead of high fructose corn syrup contradicts the terms of the written agreement and is therefore inadmissible. For these reasons, answer choices A, B, and D are incorrect.

After the death of a farmer, the executor of his estate held an auction sale of his farm equipment. The executor specified that she reserved the right to withdraw any item from the sale. A neighbor placed a bid on a tractor. The bid was acknowledged by the auctioneer. Before another bid was placed or the auctioneer announced the completion of the sale, the neighbor informed the auctioneer that he was withdrawing his bid. Must the auctioneer permit the neighbor to withdraw his bid? Answers: A. No, the auctioneer may, but is not required to, accept the withdrawal of a bid. B. No, because there is no right to withdraw a bid. C. Yes, because the seller retained a right to withdraw from the sale.

Answer choice D is correct. A bidder may retract his bid until the auctioneer announces the completion of the sale. Here, the neighbor informed the auctioneer that he was withdrawing his bid before the auctioneer announced the completion of the sale. Therefore, the auctioneer must permit the neighbor to withdraw his bid on the tractor. Answer choice A is incorrect because the option of withdrawing a bid lies with the bidder, not the auctioneer. As with any other revocation of an offer, the bidder must convey his withdrawal to the auctioneer. So long as the withdrawal is communicated to the auctioneer prior to the auctioneer announcing the completion of the sale, the auctioneer must allow the bidder to withdraw his bid. Accordingly, answer choice B is also incorrect. Answer choice C is incorrect. Regardless of whether the seller does or does not have the right to withdraw an item from the sale at the auction (i.e., the sale is with or without reserve), the bidder retains the right to withdraw a bid.

The owner of a retail clothing store regularly displayed for-sale works by local artists on a wall in the store. An art collector who came into the store inquired about purchasing a particular work for display at his home. The two agreed upon a price, but the collector was not ready to commit to purchasing it immediately. Confident that the collector would purchase the work, the owner promised in a signed writing to sell the work to the collector at the agreed-upon price at any time before the end of the month. On the last day of the month, the collector sent the owner a check for the agreed upon price, which the owner received on the following day. If the owner returns the collector's check and refuses to sell the artwork to the collector, which of the following best supports the owner's position that a contract had not been formed? Answers: A. The firm-offer rule is not applicable because the owner was not a merchant with respect to the work of art. B. The firm-offer rule is not applicable because the collector was not a merchant with respect to the work of art. C. The collector could not accept the owner's offer by mailing a check. D. The collector's acceptance of the owner's offer was not timely.

Answer choice D is correct. Although under the mailbox rule an acceptance is effective upon dispatch, the mailbox rule does not apply when the offer is irrevocable. Instead, in such case, the acceptance must be received before the offer expires. Here, the owner promised to hold her offer open until the end of the month. In order to be effective, the acceptance had to be received by the owner before the end of the month. therefore, the best argument in favor of the owner's position is that the collector failed to accept the offer by the last day of the month, because the check arrived a day later. Answer choice A is incorrect because the owner's signed written promise did create an irrevocable offer under the UCC firm-offer rule. While the rule only applies to offers made by merchants, the definition of merchant under the firm offer rule includes not only a person who regularly deals in the type of goods that are the subject of the offer (here, artwork) or otherwise by his occupation holds himself out as having knowledge or skill peculiar to the practices or goods involved in the transaction, but also any businessperson when the transaction is of a commercial nature. Answer choice A and B both indicate that the firm offer rule is inapplicable, which is not true under this definition. Answer choice B is also incorrect because the firm offer rule looks at the status of the offeror as a merchant, not the offeree; it does not require that the offeree be a merchant. Consequently, the firm-offer rule applied due to the owner's status as a merchant. Answer choice C is incorrect because, unless the offer specifies otherwise, an offer may generally be accepted by either a promise or a performance. Consequently, the collector's payment of the purchase price could constitute an acceptance.

At an auction of a rare automobile that had been owned by an infamous criminal, two individuals, a museum owner and a private collector, became involved in a bidding war. After a protracted exchange of bids, the museum owner declined the auctioneer's invitation to bid a higher amount than the private collector had. However, as the auctioneer was bringing down his hammer to announce the sale of the automobile to the private collector, the museum owner called out a higher bid before the hammer struck the sounding block on the auctioneer's desk. The auctioneer acknowledged the museum owner's bid and called for other bids. A third party placed a bid, which unmatched by either the museum owner or the private collector, was subsequently accepted by the auctioneer as the winning bid. The museum owner and private collector challenged the third party's ownership of the automobile. Who is the rightful owner of the automobile? Answers: A. The private collector, based on the private collector's last bid before the fall of the hammer. B. The private collector, based on the museum owner's refusing the auctioneer's invitation to bid prior to the fall of the hammer. C. The museum owner, based on the museum owner's bid made while the hammer was falling. D. The third party.

Answer choice D is correct. An auction sale is complete when the auctioneer announces its end, such as by the fall of the auctioneer's hammer or in any other customary way. When a bid is made contemporaneously with the falling of the hammer, the auctioneer may, at his discretion, treat the bid as continuing the bidding process or declare the sale completed at the fall of the hammer. In this case, the museum owner made his bid contemporaneously with the falling of the hammer. The auctioneer chose, at his discretion, to treat the museum owner's bid as continuing the bidding process. The third party subsequently made a bid that was unmatched. Therefore, because the third party's bid was accepted by the auctioneer as the winning bid, the third party is entitled to the automobile. Answer choice A is incorrect. Although the auctioneer could have elected to award the automobile to the private collector based on the collector's last bid made before the fall of the auctioneer's hammer, the auctioneer was not required to do so. Answer choice B is incorrect. Although the museum owner did decline the auctioneer's invitation to bid prior to the fall of the hammer, this failure to bid at that time did not preclude the auctioneer from recognizing the museum owner's subsequent bid. Answer choice C is incorrect because, although the auctioneer had the option to recognize the museum owner's bid that was made while the hammer was falling, the auctioneer could not automatically accept this bid as the final bid. Instead, having recognized this bid, the auctioneer was required to treat it as a continuation of the bidding.

On April 1, a buyer and a seller executed a written contract for the sale of an antique car for $40,000, delivery on May 1. As they each signed the contract, the buyer orally reminded the seller that the buyer's duty to purchase the car was conditioned on his ability to get approval for a loan by April 20 to fund the purchase. The seller orally agreed, though the condition was not noted in the written contract. When the seller contacted the buyer to execute the sale on May 1, he discovered that the buyer attempted but failed to get a loan and cannot afford to purchase the car. The buyer refused to honor the contract. The contract contains a clause indicating that it is a total integration of the parties' agreement. If the seller sues the buyer for breach of contract, will the court admit the evidence of the oral condition regarding the buyer's approval for a loan? Answers: A. No, because the oral agreement contradicts the terms of the written contract. B. No, because the written contract is a complete integration of the agreement between the parties. C. Yes, because the oral agreement was a distinct and separate contract. D. Yes, as proof of a condition precedent to the buyer's obligation under the contract.

Answer choice D is correct. Parol evidence may be admitted to prove a condition precedent to the existence of the contract. Therefore, the oral agreement between the buyer and seller made when they signed the contract is admissible. Answer choice A is incorrect. The parol evidence rule generally prevents a party to a written contract from presenting prior, extrinsic evidence that contradicts the terms of the contract as written. However, the parol evidence rule does not apply to evidence of a condition precedent to a party's obligation to perform. Additionally, it does not appear that the additional term evidenced by the oral agreement contradicts the written contract. Answer choice B is incorrect because evidence of a condition precedent to an obligation under a contract is admissible even if the written contract is a complete integration. Answer choice C is incorrect because the oral agreement was merely a condition precedent to an obligation under the written contract, not a separate contract.

A collector agreed to sell his collection of authentic extras' costumes from a cult classic 80's show to a costume store for $10,000, payable one month after the collection was delivered to the store via a third-party carrier. Due to the time and expense that went into accumulating and repairing the costumes, the collector expected a $2,000 profit. The costumes suffered minor water damage in transit, and the store immediately notified the collector that it was rejecting the collection and would hold the collection until the collector picked them up. The collector told the store that he would look for a new buyer and would pick up the collection in a few weeks. The collector quickly found another buyer willing to pay the original contract price. However, before the collector retrieved the costume collection, the store sold and delivered the costumes to a theater company who knowingly accepted the costumes despite the water damage. The theater company paid the store $15,000 for the collection, which the store retained. If the store's sale of the costume was NOT an acceptance, what is highest value remedy available to the collector? Answers: A. $2,000, the collector's lost profit. B. $5,000, the difference between market price and contract price. C. $10,000, the collector-store contract price. D. $15,000, damages for conversion.

Answer choice D is correct. The store's rejection of the collection was proper under the perfect tender rule, but the store's selling the collection to the theater company constituted conversion. The remedy for conversion is the fair market value of the goods at the time of the conversion of the collection. The $15,000 received by the store from the sale of the collection reflects the collection's fair market value at the time of the conversion. Answer choice A is incorrect because the collector is not a volume seller; consequently, lost profit damages are not available to him. Answer choice B is incorrect because the difference between market and contract prices is an improper measure of damages in cases of conversion. Answer choice C is incorrect because the collector is not limited to the contract price in ascertaining the value of the collection; instead, he is entitled to fair market price of the costumes.


Related study sets

Industrial Revolution Project - Light Bulb

View Set

Course 1 - QUIZ: Requirements and Architecture

View Set

ch. 14, 15 & 16 Career counseling

View Set

Interactive Animation: Stream Processes: Oxbow Lakes and Floodplain Development

View Set

RSM: Introduction to Business Midterm

View Set